Consider the matrix \[\left[ {\begin{array}{*{20}{c}} 5&{ – 1} \\ 4&1 \end{array}} \right]\] . Which one of the following statements is TRUE for the eigen values and eigen vectors of this matrix? A. Eigen value 3 has a multiplicity of 2, and only one independent eigen vector exists B. Eigen value 3 has a multiplicity of 2, and two independent eigen vector exists C. Eigen value 3 has a multiplicity of 2, and no independent eigen vector exists D. Eigen value are 3 and -3, and two independent eigen vectors exist

Eigen value 3 has a multiplicity of 2, and only one independent eigen vector exists
Eigen value 3 has a multiplicity of 2, and two independent eigen vector exists
Eigen value 3 has a multiplicity of 2, and no independent eigen vector exists
Eigen value are 3 and -3, and two independent eigen vectors exist

The correct answer is $\boxed{\text{B}}$.

To find the eigenvalues and eigenvectors of a matrix, we can use the following formula:

$$\lambda v = A v$$

where $\lambda$ is an eigenvalue, $v$ is an eigenvector, and $A$ is the matrix.

In this case, we have the matrix $A = \left[ {\begin{array}{*{20}{c}} 5&{ – 1} \ 4&1 \end{array}} \right]$.

To find the eigenvalues, we can solve the equation $Av = \lambda v$.

We can do this by first finding the characteristic polynomial of $A$, which is given by:

$$| A – \lambda I |$$

In this case, the characteristic polynomial is:

$$| \left[ {\begin{array}{{20}{c}} 5&{ – 1} \ 4&1 \end{array}} \right] – \lambda \left[ {\begin{array}{{20}{c}} 1&0 \ 0&1 \end{array}} \right] | = \left[ {\begin{array}{*{20}{c}} 5 – \lambda &{ – 1} \ 4&1 – \lambda \end{array}} \right]$$

We can then solve the equation $| A – \lambda I | = 0$ to find the eigenvalues.

In this case, we get the eigenvalues $\lambda = 3$ and $\lambda = -3$.

To find the eigenvectors, we can substitute each eigenvalue into the equation $Av = \lambda v$ and solve for $v$.

In this case, we get the following eigenvectors:

$$v_1 = \left[ {\begin{array}{*{20}{c}} 1 \ 1 \end{array}} \right]$$

and

$$v_2 = \left[ {\begin{array}{*{20}{c}} -1 \ 1 \end{array}} \right]$$

We can see that the eigenvalue $\lambda = 3$ has multiplicity 2, since there are two linearly independent eigenvectors associated with it.

Therefore, the correct answer is $\boxed{\text{B}}$.

Exit mobile version